• Nie Znaleziono Wyników

On the number of good approximations of algebraic numbers by algebraic numbers of bounded degree

N/A
N/A
Protected

Academic year: 2021

Share "On the number of good approximations of algebraic numbers by algebraic numbers of bounded degree"

Copied!
26
0
0

Pełen tekst

(1)

LXXXIX.2 (1999)

On the number of good approximations of

algebraic numbers by algebraic numbers of bounded degree

by

Helmut Locher (Marburg)

1. Introduction. Let α be an algebraic number. Roth’s celebrated the- orem [13] says that for any δ > 0 there are only a finite number of rational approximations x/y of α with

(1.1) |α − x/y| < 1/y

2+δ

, y > 0.

In this paper we consider approximations of α by algebraic numbers of bounded degree. More precisely, let d ∈ N and suppose µ > 2. We look for solutions in algebraic numbers β of degree ≤ d of the inequality

(1.2) |α − β| < H

0

(β)

−µ

,

where H

0

(β) denotes the maximum modulus of the coefficients of the mini- mal defining polynomial of α over Z. For rational β, say β = x/y, we have H

0

(β) = max{|x|, |y|} and hence for d = 1 the inequality (1.2) is essentially equivalent to (1.1).

Wirsing [19] proved that (1.2) has for

(1.3) µ > 2d

only a finite number of solutions.

As a consequence of his famous subspace theorem W. M. Schmidt [15]

was able to prove the best possible result ([16], p. 278): (1.2) has for

(1.4) µ > d + 1

only a finite number of solutions.

Unfortunately, the underlying method of Thue–Siegel–Roth is ineffective in the sense that it does not provide upper bounds for y or H

0

(β) respec- tively. However, it allows giving an explicit upper bound for the number of x/y ∈ Q satisfying (1.1). A first result was proved by Davenport and Roth ([3], 1955). This bound was improved by Bombieri and van der Poorten ([1], 1987) and independently by Luckhardt ([10], 1989) using the modified proof

1991 Mathematics Subject Classification: Primary 11J68.

[97]

(2)

of Roth’s Theorem presented by Esnault and Viehweg ([4], 1984). The latest results are due to Evertse ([7], 1996, [8], 1998).

It is the purpose of this paper to prove such a quantitative result of Wirsing’s theorem.

To state our theorems we have to define the height of an algebraic num- ber. Let K be a number field and M (K) its set of places. For v ∈ M (K) denote by | · |

v

the associated absolute value, normalized so that on Q we have | · |

v

= | · | (standard absolute value) if v is archimedean, whereas for v non-archimedean |p|

v

= p

−1

if v lies above the rational prime p. We put

k · k

v

= | · |

[Kv v:Qp]/[K:Q]

,

where K

v

denotes the completion of (K, |·|

v

) and Q

p

denotes the completion of (Q, | · |

p

). We also denote the unique extensions of | · |

v

and k · k

v

to K

v

by | · |

v

and k · k

v

respectively. For x ∈ K we define the height of x by

(1.5) H(x) = Y

v∈M (K)

max{1, kxk

v

}.

Let | · | denote the standard absolute value of the complex numbers C, and Q the algebraic closure of Q in C. For any positive number x we define log

+

x = log x if x ≥ e and 1 otherwise.

The following is a quantitative version of the result (1.3) of Wirsing ([19], Theorem 1).

Theorem 1. Let 0 < δ ≤ 1, d ∈ N and α be an algebraic number of degree f . Consider the inequality

(1.6) |α − β| < H(β)

−2d2−δ

to be solved in elements β ∈ Q with

(1.7) deg β ≤ d.

(i) There are at most

e

26

· d

15

log(6f )

δ

5

log d log(6f ) δ

solutions β ∈ Q of (1.6) and (1.7) with H(β) ≥ max{4

4d2

, H(α)}.

(ii) There are at most

log

+

log H(α)

log(1 + δ/(4d

2

)) + 2

15d2

δ

solutions β ∈ Q of (1.6) and (1.7) with H(β) < max{4

4d2

, H(α)}.

We suppose every number field to be embedded in Q and every valuation

of the number field to be extended to Q. The following generalizes Theorem

1 to include non-archimedean primes.

(3)

Theorem 2. Let 0 < δ ≤ 1, d ∈ N and F/K be an extension of number fields of degree f . Let S be a finite set of places of K of cardinality s.

Suppose that for each v ∈ S we are given a fixed element α

v

∈ F . Let H be a real number with H ≥ H(α

v

) for all v ∈ S. Consider the inequality

(1.8) Y

v∈S

min{1, kα

v

− βk

v

} < H(β)

−2d2−δ

to be solved in elements β ∈ Q with

(1.9) [K(β) : K] ≤ d.

Then there are at most

e

7s+19

· d

2s+13

log(6f )

δ

s+4

log d log(6f ) δ solutions β ∈ Q of (1.8) and (1.9) with

(1.10) H(β) ≥ max{H, 4

4d2

}.

We have claimed above that Theorems 1 and 2 are quantitative versions of Wirsing’s result (1.3). But in our theorems we have the exponent 2d

2

instead of 2d. The reason is that our height H(·) as defined in (1.5) is normalized in a different way than the height H

0

(·) in (1.2). For algebraic numbers β of degree ≤ d we have ([17], Chapter I, Lemma 7B)

H(β)

d



d

H

0

(β) 

d

H(β)

d

.

Therefore we get an additional factor d in the exponent. For the height H(·) the best possible exponent in (1.6) and (1.8) would be d(d + 1).

To prove the best possible result Schmidt uses an induction argument which depends upon his subspace theorem. It is not clear how this argument can be used to obtain a quantitative result.

Independently, J.-H. Evertse [8] also proved a quantitative version of Wirsing’s theorem (1.3). Moreover, he gave an explicit upper bound for the number of solutions of a more general problem considered by Wirsing [19].

His upper bounds for (1.3) are similar to ours.

2. The auxiliary polynomial

2.1. A generalization of the index. Let P be a non-zero polynomial in m variables X

1

, . . . , X

m

with complex coefficients. Roth [13] introduced the index of a polynomial at a certain point to measure to what extent the polynomial vanishes at that point. In this section we will define a different measure for this need. It was introduced by W. M. Schmidt ([18], p. 139).

Let α ∈ C

m

and r ∈ N

m

. We write P in the form P (X

1

, . . . , X

m

) = X

i

a

i

(α)(X

1

− α

1

)

i1

. . . (X

m

− α

m

)

im

(4)

with i = (i

1

, . . . , i

m

) and unique coefficients a

i

(α). Let M be a subset of R

m

and put

k

α,r

(P ) = {(i

1

/r

1

, . . . , i

m

/r

m

) : a

i

(α) 6= 0}.

We say P is M-centered at α with respect to r if k

α,r

(P ) ⊆ M .

2.2. Estimation of volumes. Suppose 0 ≤ γ ≤ 1 and ε > 0. Let m ∈ N.

We put

ξ

γ

(x) = |{h ∈ {1, . . . , m} : 0 ≤ x

h

≤ γ}| = X

m h=1

χ

[0,γ]

(x

h

),

where χ

[0,γ]

denotes the characteristic function of the closed interval [0, γ].

The sets

M

ε

(m, γ) = {x ∈ [0, 1]

m

: ξ

γ

(x) ≤ m(γ + ε)}, M

ε

(m) = \

γ∈[0,1]

M

ε

(m, γ) are the main objects of this section. We always consider the complement of M

ε

(m, γ) and M

ε

(m) with respect to [0, 1]

m

. More precisely, we put

M

εc

(m, γ) = [0, 1]

m

− M

ε

(m, γ) and M

εc

(m) = [0, 1]

m

− M

ε

(m).

Lemma 2.1. Suppose 0 ≤ γ ≤ 1, ε > 0 and let m ∈ N. Then

(2.1) \

Mεc(m,γ)

dx ≤ e

−γ(1−γ)ε2m

.

The line of the proof is the same as the proof of [16], Chapter V, Lemma 4C.

P r o o f. The integral on the left-hand side of (2.1) exists, since the bound- ary of M

εc

(m, γ) lies in a finite union of hyperplanes. For all x ∈ M

εc

(m, γ) we have ξ

γ

(x) − mγ > mε. Therefore

(2.2) e

γε2m

\

Mεc(m,γ)

dx

\

Mεc(m,γ)

e

γε(ξγ(x)−mγ)

dx = \

Mεc(m,γ)

e

γε((Pmh=1χ[0,γ](xh))−mγ)

dx

= \

Mεc(m,γ)

e

γεPmh=1[0,γ](xh)−γ)

dx ≤ \

[0,1]m

Y

m h=1

e

γε(χ[0,γ](xh)−γ)

dx

=



1

\

0

e

γε(χ[0,γ](x)−γ)

dx



m

.

(5)

Note that e

y

≤ 1 + y + y

2

for |y| ≤ 1. Hence we get (2.3)

1

\

0

e

γε(χ[0,γ](x)−γ)

dx

1

\

0

(1 + γε(χ

[0,γ]

(x) − γ) + γ

2

ε

2

[0,γ]

(x) − γ)

2

) dx

≤ 1 + γε

1

\

0

[0,γ]

(x) − γ) dx + γ

2

ε

2

= 1 + γ

2

ε

2

. (2.2) and (2.3) together give

e

γε2m

\

Mεc(m,γ)

dx ≤ (1 + γ

2

ε

2

)

m

≤ e

γ2ε2m

and the lemma follows.

Lemma 2.2. Suppose 0 < ε ≤ 2/3 and let m ∈ N. Then

\

Mεc(m)

dx < 2e

−(mε3/16+log ε)

.

P r o o f. In analogy to Lemma 2.1 the integral on the left-hand side exists since the boundary lies in a finite union of hyperplanes. We put

n = (

2

ε

(1 − ε) if

2ε

(1 − ε) ∈ N,



2

ε

(1 − ε) 

+ 1 otherwise;

γ

i

= i · ε 2



1 ≤ i ≤

 2

ε (1 − ε)



, γ

n

= 1 − ε.

For every γ ∈ [0, 1 − ε] there exists an i ∈ {1, . . . , n} with

(2.4) γ

i

− ε/2 ≤ γ ≤ γ

i

.

Next we show that

(2.5) M

ε

(m) ⊇

\

n i=1

M

ε/2

(m, γ

i

).

Trivially, ξ

γ

(x) ≤ m, and so we have M

ε

(m) = \

γ∈[0,1]

M

ε

(m, γ) = \

γ∈[0,1−ε]

M

ε

(m, γ).

Now let γ ∈ [0, 1 − ε]. Take i ∈ {1, . . . , n} satisfying (2.4). Since ξ

γ

(x) is non-decreasing in γ, for all x ∈ T

n

j=1

M

ε/2

(m, γ

j

) we get

ξ

γ

(x) ≤ ξ

γi

(x) ≤ m(γ

i

+ ε/2) ≤ m(γ + ε/2 + ε/2) = m(γ + ε)

and hence x ∈ M

ε

(m, γ). Thus we have verified (2.5).

(6)

From (2.5) we get by De Morgan’s formulae M

εc

(m) ⊆ S

n

i=1

M

ε/2c

(m, γ

i

).

We now apply Lemma 2.1 to get

\

Mεc(m)

dx ≤ \

Sn

i=1Mε/2c (m,γi)

dx ≤ X

n i=1

e

−γi(1−γi2m/4

.

Since

1≤i≤n

min γ

i

(1 − γ

i

) = ε 2

 1 − ε

2



ε

4 and n ≤

 2 ε (1 − ε)

 + 1 ≤

 2 ε



− 1 ≤ 2 ε we conclude

\

Mεc(m)

dx ≤ ne

−ε3m/16

2

ε e

−ε3m/16

≤ 2e

−(ε3m/16+log ε)

.

The following lemma is one of the main reasons for the exponent 2d

2

. Lemma 2.3 ([18], Lemma 7.2.1). Let I

1

, . . . , I

D

be subsets of {1, . . . , m}

and let e d ∈ N with P

D

k=1

|I

k

| ≥ Dm/ e d. Then X

D

k=1

inf n X

h∈Ik

x

h

: x ∈ M

ε

(m) o

Dm

2 e d

2

(1 − 2ε e d

2

).

Lemma 2.4. Let r

1

, . . . , r

m

∈ N. The number of tuples i ∈ Z

m

with 0 ≤ i

h

≤ r

h

(1 ≤ h ≤ m) and (i

1

/r

1

, . . . , i

m

/r

m

) 6∈ M

ε

(m) is

r

1

. . . r

m

\

Mεc(m)

dx + O

m

 r

1

. . . r

m

min

1≤h≤m

r

h

 .

P r o o f. We put ξ

γ,r

(x) = |{h ∈ {1, . . . , m} : x

h

/r

h

≤ γ}| and M = {x ∈ [0, r

1

] × . . . × [0, r

m

] : ξ

γ,r

(x) ≤ m(γ + ε), ∀γ ∈ [0, 1]}, M

c

= {x ∈ [0, r

1

] × . . . × [0, r

m

] : ∃γ ∈ [0, 1] : ξ

γ,r

(x) > m(γ + ε)}.

Observe that T

Mc

dx = r

1

. . . r

m

T

Mεc(m)

dx. We denote by G

c

the set of integer points of M

c

, thus

G

c

= {i ∈ Z

m

: (i

1

/r

1

, . . . , i

m

/r

m

) 6∈ M

ε

(m), 0 ≤ i

h

≤ r

h

, 1 ≤ h ≤ m}.

For i ∈ Z

m

we put

Q

i

= [i

1

, i

1

+ 1] × . . . × [i

m

, i

m

+ 1].

Now we can write the assertion as

\

S

i∈GcQi

dx = \

Mc

dx + O

m

 r

1

. . . r

m

min

1≤h≤m

r

h



.

(7)

For x ∈ M

c

it follows that ([x

1

], . . . , [x

m

]) ∈ M

c

and hence x ∈ S

i∈Gc

Q

i

. In other words, M

c

S

i∈Gc

Q

i

. Therefore it suffices to show

(2.6) \

S

i∈GcQi−Mc

dx = O

m

 r

1

. . . r

m

min

1≤h≤m

r

h

 .

We have [

i∈Gc

Q

i

− M

c

= {x ∈ [0, r

1

] × . . . × [0, r

m

] :

∃γ ∈ [0, 1] : ξ

γ,r

(([x

1

], . . . , [x

m

])) > m(γ + ε)} ∩ M.

Let x ∈ S

i∈Gc

Q

i

− M

c

. There exists some e γ ∈ [0, 1] with (2.7) |{h ∈ {1, . . . , m} : [x

h

]/r

h

≤ e γ}| > m(e γ + ε).

On the other hand, for all γ ∈ [0, 1] we have

(2.8) |{h ∈ {1, . . . , m} : x

h

/r

h

≤ γ}| ≤ m(γ + ε).

Observe that for all permutations π of {1, . . . , m}, (2.9) (x

π(1)

, . . . , x

π(m)

) ∈ [

i∈Gc

Q

i

− M

c

. Thus additionally we can assume

(2.10) x

1

/r

1

≤ . . . ≤ x

m

/r

m

and therefore we also have

(2.11) [x

1

]/r

1

≤ . . . ≤ [x

m

]/r

m

.

We put e h = |{h ∈ {1, . . . , m} : [x

h

]/r

h

≤ e γ}|. Then (2.7) and (2.11) together give

(2.12) e h > m(e γ + ε) ≥ m([x

eh

]/r

he

+ ε).

If we choose γ = x

eh

/r

eh

, then (2.10) and (2.8) imply

(2.13) e h ≤ |{h ∈ {1, . . . , m} : x

h

/r

h

≤ x

eh

/r

eh

}| ≤ m(x

he

/r

he

+ ε).

The combination of (2.13) and (2.12) gives

(2.14) r

eh

(e h/m − ε) ≤ x

eh

< r

he

(e h/m − ε) + 1.

The value e h depends on x, but the possible values of e h range between 1 and m, since e h is positive. As S

i∈Gc

Q

i

− M

c

⊆ [0, r

1

] × . . . × [0, r

m

] we finally conclude from (2.14) that

\

{x∈S

i∈GcQi−Mc:x1/r1≤...≤xm/rm}

dx ≤ X

m eh=1

r

1

. . . r

m

r

eh

= O

m

 r

1

. . . r

m

min

1≤h≤m

r

h

 .

Now (2.6) follows immediately using (2.9).

(8)

Lemma 2.5. Suppose ε > 0. Let P ∈ C[X

1

, . . . , X

m

], α ∈ C

m

and r ∈ N

m

. Let j ∈ Z

m

with 0 ≤ j

h

≤ r

h

(1 ≤ h ≤ m) and j

1

/r

1

+ . . . + j

m

/r

m

≤ ε.

Suppose P is M

ε

(m)-centered at α with respect to r. Then

j1+...+jm

∂X

1j1

. . . ∂X

mjm

P is M

(m)-centered at α with respect to r.

P r o o f. Since j

h

/r

h

≤ ε for all h ∈ {1, . . . , m} the lemma is an easy consequence of the definition of the set M

ε

(m) .

2.3. Heights and Siegel’s Lemma. Let K be a number field and M (K) its set of places. Let n ∈ N. For x ∈ K

n

and v ∈ M (K) we put

|x|

v

= max{|x

1

|

v

, . . . , |x

n

|

v

} and kxk

v

= |x|

[Kv v:Qp]/[K:Q]

. If v is archimedean we put

|x|

v,E

= (|x

1

|

2v

+ . . . + |x

n

|

2v

)

1/2

and kxk

v,E

= |x|

[Kv,Ev:Qp]/[K:Q]

. The height and the euclidean height of x ∈ K

n

are defined by

H(x) = Y

v∈M (K)

kxk

v

, H

E

(x) =

 Y

v∈M (K) v|∞

kxk

v,E

 Y

v∈M (K) v

-

kxk

v

.

We have H(x) ≤ H

E

(x) ≤

n H(x). The height of a polynomial is defined as the height of its coefficient vector. We use the notation

i

= 1

i

1

! . . . i

m

!

i1+...+im

∂X

1i1

. . . ∂X

mim

.

Let P ∈ Q[X

1

, . . . , X

m

] have degree ≤ r

h

in X

h

(1 ≤ h ≤ m). Let j ∈ Z

m

with j

h

≥ 0 (1 ≤ h ≤ m). We have

(2.15) H(∆

j

P ) ≤ 2

r1+...+rm

H(P ).

Finally, we are able to construct the auxiliary polynomial.

Lemma 2.6. Suppose 0 < ε < 1. Let F/K be an extension of number fields of degree f , let α

1

, . . . , α

s

∈ F and m ∈ N. Suppose

m ≥ 16

ε

3

(log(6sf ) + log ε

−1

).

There is a constant R = R(m) such that for all r = (r

1

, . . . , r

m

) ∈ N

m

with r

h

≥ R (1 ≤ h ≤ m) there exists a non-zero polynomial P ∈ K[X

1

, . . . , X

m

] such that

(i) deg

Xh

P ≤ r

h

(1 ≤ h ≤ m);

(ii) P is M

ε

(m)-centered at the points α

k

= (α

k

, . . . , α

k

) (1 ≤ k ≤ s)

with respect to r;

(9)

(iii) H(P ) ≤ C(F )(4H)

r1+...+rm

, where H = max{H(α

1

), . . . , H(α

s

)}

and C(F ) denotes a constant depending only on F . P r o o f. We put N = (r

1

+ 1) . . . (r

m

+ 1) and

M = |{i ∈ Z

m

: (i

1

/r

1

, . . . , i

m

/r

m

) 6∈ M

ε

(m), 0 ≤ i

h

≤ r

h

, 1 ≤ h ≤ m}|.

Let P ∈ K[X

1

, . . . , X

m

] with (i). We need to determine the coefficients of P such that (ii) and (iii) hold. (ii) says that

(2.16)

i

P (α

k

) = 0

for all i ∈ Z

m

with (i

1

/r

1

, . . . , i

m

/r

m

) 6∈ M

ε

(m), 0 ≤ i

h

≤ r

h

, 1 ≤ h ≤ m and 1 ≤ k ≤ s. (2.16) is a system of linear equations, where the unknowns are the coefficients of P . To solve (2.16) we will apply Siegel’s Lemma in the form given by Bombieri and Vaaler [2].

Lemma 2.4 says

M = r

1

. . . r

m

\

Mεc(m)

dx + O

m

 r

1

. . . r

m

min

1≤h≤m

r

h



and therefore it follows from Lemma 2.2 that M

N = \

Mεc(m)

dx + 1

(r

1

+ 1) . . . (r

m

+ 1) · O

m

 r

1

. . . r

m

min

1≤h≤m

r

h



< 2e

−(mε3/16+log ε)

+ 1

(r

1

+ 1) . . . (r

m

+ 1) · O

m

 r

1

. . . r

m

min

1≤h≤m

r

h

 . Hence for large r

1

, . . . , r

m

we get

(2.17) M/N < 3e

−(mε3/16+log ε)

. By assumption,

m ≥ 16

ε

3

(log(6sf ) + log ε

−1

) = 16

ε

3

log(6sf /ε).

This is equivalent to

(2.18) 3sf e

−(mε3/16+log ε)

≤ 1/2.

The inequalities (2.17) and (2.18) together give

(2.19) sf M < N sf 3e

−(mε3/16+log ε)

≤ N/2.

If we denote by A the matrix corresponding to (2.16), then by [2], Theorem 12 and (2.19) we get a non-zero polynomial P ∈ K[X

1

, . . . , X

m

] satisfying (i), (2.16) and

H(P ) ≤ C(F )( max

a row of A

H

E

(a))

sf M /(N −sf M )

≤ C(F ) max

a row of A

H

E

(a),

where C(F ) denotes a constant only depending on F . By standard estimates

we know that H

E

(a) ≤ (4H)

r1+...+rm

and the lemma follows.

(10)

3. Roth’s Lemma. The essential ingredient to Roth’s Theorem in [13]

is the so-called Roth’s Lemma. We quote its version proved by J. H. Evertse [6], which is a quantitative improvement on the original. J. H. Evertse proved this result by using Faltings’ Product Theorem [9].

Let P be a non-zero polynomial in unknowns X

1

, . . . , X

m

with complex coefficients. Let α ∈ C

m

and r ∈ N

m

. We define, as in [13],

Ind

α,r

P

= min{i

1

/r

1

+ . . . + i

m

/r

m

: ∆

i

P (α) 6= 0, i ∈ Z

m

, i

h

≥ 0, 1 ≤ h ≤ m}

and say that P has index Ind

α,r

P at α with respect to r.

Proposition 3.1 ([6], Theorem 3). Let m be an integer ≥ 2, let r = (r

1

, . . . , r

m

) be a tuple of positive integers, let P ∈ Q[X

1

, . . . , X

m

] be a non- zero polynomial of degree ≤ r

h

in X

h

for h = 1, . . . , m and let 0 < ε ≤ m+1 be such that

(3.1) r

h

/r

h+1

≥ 2m

3

for h = 1, . . . , m − 1.

Further , let β

1

, . . . , β

m

be algebraic numbers with

(3.2) H

E

((1, β

h

))

rh

> {e

r1+...+rm

H

E

(P )}

(3m3/ε)m

(1 ≤ h ≤ m).

Then Ind

β,r

P < ε.

4. A quantitative result. Suppose 0 < ε < 1. Let F/K be an extension of number fields of degree f . Let S be a finite subset of M (K) of cardinality s. Suppose that for each v ∈ S we are given fixed elements α

v

∈ F . Suppose H(α

v

) ≤ H (v ∈ S). Let m ∈ N with m ≥ (16/ε

3

)(log(6sf ) + log ε

−1

).

Under these assumptions the hypotheses of Lemma 2.6 are satisfied.

Let R = R(m) be the constant given by Lemma 2.6. Suppose r

h

≥ R (1 ≤ h ≤ m). Then there is a polynomial P with

(4.1) P ∈ K[X

1

, . . . , X

m

], P 6= 0;

(4.2) deg

Xh

P ≤ r

h

(1 ≤ h ≤ m);

(4.3) P is M

ε

(m)-centered with respect to r

at the points α

v

= (α

v

, . . . , α

v

) (v ∈ S);

(4.4) H(P ) ≤ C(4H)

r1+...+rm

, where C = C(F ) is a constant just depending on F .

Lemma 4.1. Suppose 0 < δ ≤ 1, d ∈ N and 0 < ε ≤ δ/(20d

4

). Let Γ be a tuple of non-negative integers with

X

v∈S

Γ

v

= 1 − δ/(24d

2

).

(11)

Suppose there are elements β

1

, . . . , β

m

∈ Q satisfying [K(β

h

) : K] ≤ d,

H(β

1

)

r1

≤ H(β

h

)

rh

≤ H(β

1

)

(1+ε)r1

(1 ≤ h ≤ m), (4.5)

v

− β

h

k

v

< H(β

h

)

−Γv(2d2+δ)

(1 ≤ h ≤ m, v ∈ S) (4.6)

and

(4.7) H(β

h

)

ε/2

≥ max{C

1/r1

, 2

7

H

3f s

} (1 ≤ h ≤ m).

Then Ind

β,r

P > ε.

P r o o f. Let j ∈ Z

m

with 0 ≤ j

h

≤ r

h

, 1 ≤ h ≤ m and (4.8) j

1

/r

1

+ . . . + j

m

/r

m

≤ ε.

Put

(4.9) T (X) = X

i

a

i

X

1i1

. . . X

mim

= ∆

j

P (X).

We have to show

(4.10) T (β) = 0.

First we establish an inequality for the height of T . From (2.15), (4.2), (4.4) and (4.7) we get

(16H

2f s

)

r1+...+rm

H(T ) ≤ (2

5

H

2f s

)

r1+...+rm

H(P ) ≤ C(2

7

H

3f s

)

r1+...+rm

≤ C

 Y

m

h=1

H(β

h

)

rh



ε/2

.

By (4.7) we have C ≤ H(β

1

)

r1ε/2

Q

m

h=1

H(β

h

)

rhε/2

and therefore (4.11) (16H

2f s

)

r1+...+rm

H(T ) ≤

 Y

m

h=1

H(β

h

)

rh



ε

.

We will need (4.11) later on.

Put E = K(β

1

, . . . , β

m

). We denote by E ,→ K

K v

the set of K-embeddings of E into K

v

, i.e. the homomorphisms of E in K

v

which are the identity on K.

For each place w of E which lies over v of K, there exists a λ ∈ E ,→ K

K v

with

|a|

w

= |λ(a)|

v

for all a ∈ E. There are in fact [E

w

: K

v

] such embeddings.

With these notations the product formula reads Y

p∈M (Q)

Y

w∈M (E) w|p

|x|

[Eww:Qp]

= Y

p∈M (Q)

Y

v∈M (K) v|p

Y

λ∈E,→KK v

|λ(x)|

[Kv v:Qp]

= 1

(12)

for all x ∈ E

. From (4.1) and (4.9) we know that T has coefficients in K and hence T (β) ∈ E. Therefore to prove (4.10) it suffices to show

(4.12) Y

p∈M (Q)

Y

v∈M (K) v|p

Y

λ∈E,→KK v

|T (λ(β))|

[Kv v:Qp]

< 1.

Let v ∈ M (K). In the sequel we estimate Q

λ∈E,→KK v

|T (λ(β))|

v

. Put

(4.13) κ

v

=

 1 if v | ∞, 0 if v - ∞

and r = r

1

+ . . . + r

m

. For v 6∈ S, by trivial estimates of (4.9) we get Y

λ∈E,→KK v

|T (λ(β))|

v

Y

λ∈E,→KK v



2

κvr

max

i

|a

i

|

v

Y

m h=1

|(1, λ(β

h

))|

rvh

 (4.14)

= 2

κv[E:K]r

max

i

|a

i

|

[E:K]v

Y

λ∈E,→KK v

1≤h≤m

|(1, λ(β

h

))|

rvh

.

Now, let v ∈ S. We can write (4.6) as (4.15)

v

v

) − µ

v

h

)|

v

< H(β

h

)

−Γv(2d2+δ)[K:Q]/[Kv:Qp]

(1 ≤ h ≤ m, v ∈ S), where µ

v

denotes a fixed K-embedding of Q in K

v

. Let λ ∈ E ,→ K

K v

. We expand T (X) around the point µ

v

v

) = (µ

v

v

), . . . , µ

v

v

)) in a Taylor series to get

(4.16) |T (λ(β))|

v

≤ 2

κvr

max

i

i

T (µ

v

v

)) Y

m h=1

(λ(β

h

) − µ

v

v

))

ih

v

. By trivial estimates we get

(4.17) |∆

i

T (µ

v

v

))|

v

≤ 4

κvr

max

ei

|a

ei

|

v

|(1, µ

v

v

))|

rv

. The main term we have to look at is max

i

Q

m

h=1

|λ(β

h

) − µ

v

v

)|

ivh

, where the maximum is taken over all i with ∆

i

T (µ

v

v

))

v

6= 0. By (4.3), (4.8), Lemma 2.5 and µ

v

(∆

i

T (α

v

)) = ∆

i

T (µ

v

v

)), for tuples i satisfy- ing ∆

i

T (µ

v

v

))

v

6= 0 we have (i

1

/r

1

, . . . , i

m

/r

m

) ∈ M

(m). Therefore it suffices to consider the term sup

x∈M(m)

Q

m

h=1

|λ(β

h

) − µ

v

v

)|

xvhrh

. If λ(β

h

) = µ

v

h

), we can estimate the factor satisfying (4.15) non- trivially. Hence we treat the cases λ(β

h

) = µ

v

h

) and λ(β

h

) 6= µ

v

h

) separately. Put

I

λ

= {h ∈ {1, . . . , m} : λ(β

h

) = µ

v

h

)}.

(13)

We have

(4.18) sup

x∈M(m)

Y

m h=1

|λ(β

h

) − µ

v

v

)|

xvhrh

 sup

x∈M(m)

Y

h6∈Iλ

|λ(β

h

) − µ

v

v

)|

xvhrh



×

 sup

x∈M(m)

Y

h∈Iλ

|λ(β

h

) − µ

v

v

)|

xvhrh

 . We estimate the first factor of (4.18) trivially and get

(4.19) sup

x∈M(m)

Y

h6∈Iλ

|λ(β

h

) − µ

v

v

)|

xvhrh

Y

m h=1

2

κvrh

|(1, λ(β

h

))|

rvh

|(1, µ

v

v

))|

rvh

. For the second factor of (4.18) we use (4.15) and (4.5) to get

sup

x∈M(m)

Y

h∈Iλ

|λ(β

h

) − µ

v

v

)|

xvhrh

= sup

x∈M(m)

Y

h∈Iλ

v

v

) − µ

v

h

)|

xvhrh

< sup

x∈M(m)

Y

h∈Iλ

H(β

h

)

−xhrhΓv(2d2+δ)[K:Q]/[Kv:Qp]

sup

x∈M(m)

H(β

1

)

−r1Γv(2d2+δ)[Kv :Qp][K:Q]

P

h∈Iλxh

= H(β

1

)

−r1Γv(2d2+δ)[Kv :Qp][K:Q] infx∈M2ε(m)

P

h∈Iλxh

. Taking the product over all K-embeddings of E into K

v

gives

(4.20) Y

λ∈E,→KK v

sup

x∈M(m)

Y

h∈Iλ

|λ(β

h

) − µ

v

v

)|

xvhrh

< H(β

1

)

−r1Γv(2d

2+δ)[Kv :Qp][K:Q] P

λ∈EK ,→Kv

infx∈M2ε(m)P

h∈Iλxh

. To apply Lemma 2.3 we need a lower bound for P

λ∈E,→KK v

|I

λ

|. Let δ

x,y

denote the Kronecker symbol. We have X

λ∈E,→KK v

|I

λ

| = X

λ∈E,→KK v

|{h ∈ {1, . . . , m} : λ(β

h

) = µ

v

h

)}|

= X

λ∈E,→KK v

X

1≤h≤m

δ

λ(βh),µvh)

= X

1≤h≤m

X

λ∈E,→KK v

δ

λ(βh),µvh)

= X

1≤h≤m

[E : K(β

h

)] ≥ X

1≤h≤m

[E : K]

d = m[E : K]

d .

(14)

Now we apply Lemma 2.3 to (4.20) with [E : K] in place of D and 2ε in place of ε to get

(4.21) Y

λ∈E,→KK v

sup

x∈M(m)

Y

h∈Iλ

|λ(β

h

) − µ

v

v

)|

xvhrh

< H(β

1

)

[Kv :Qp][E:Q] mr1(1−4εd2)(1+2d2δ v

. The combination of (4.18), (4.19) and (4.21) gives

(4.22) Y

λ∈E,→KK v

sup

x∈M(m)

Y

m h=1

|λ(β

h

) − µ

v

v

)|

xvhrh

<

 Y

λ∈E,→KK v 1≤h≤m

2

κvrh

|(1, µ

v

v

))|

rvh

|(1, λ(β

h

))|

rvh



× H(β

1

)

[Kv :Qp][E:Q] mr1(1−4εd2)(1+2d2δ v

< (2

κv

|(1, µ

v

v

))|

v

)

r[E:K]

 Y

λ∈E,→KK v

1≤h≤m

|(1, λ(β

h

))|

rvh



× H(β

1

)

[Kv :Qp][E:Q] mr1(1−4εd2)(1+2d2δ v

and the combination of (4.16), (4.17) and (4.22) gives

(4.23) Y

λ∈E,→KK v

|T (λ(β))|

v

< Y

λ∈E,→KK v

2

κvr

max

i

|∆

i

T (µ

v

v

))|

v

Y

m h=1

|λ(β

h

) − µ

v

v

)|

ivh

≤ 8

κv[E:K]r

max

ei

|a

ei

|

[E:K]v

|(1, µ

v

v

))|

[E:K]rv

× Y

λ∈E,→KK v

max

i

Y

m h=1

|λ(β

h

) − µ

v

v

)|

ivh

< 16

κv[E:K]r

max

i

|a

i

|

[E:K]v

|(1, µ

v

v

))|

2[E:K]rv

×

 Y

λ∈E,→KK v

1≤h≤m

|(1, λ(β

h

))|

rvh



H(β

1

)

[Kv :Qp][E:Q] mr1(1−4εd2)(1+2d2δ v

.

Finally, if we take the product over all valuations of E, then (4.14) and

(4.23) together lead to

(15)

(4.24) Y

p∈M (Q)

Y

v∈M (K) v|p

 Y

λ∈E,→KK v

|T (λ(β))|

v



[Kv:Qp]

<  Y

p∈M (Q)



16

[E:K]rPv∈M (K),v|pκv[Kv:Qp]

Y

v∈S, v|p

|(1, µ

v

v

))|

2[E:K]r[Kv v:Qp]



×  Y

p∈M (Q)

 Y

v∈M (K) v|p

max

i

|a

i

|

[Kv v:Qp]



[E:K]



×  Y

p∈M (Q)

Y

v∈M (K) v|p

 Y

λ∈E,→KK v

1≤h≤m

|(1, λ(β

h

))|

rvh



[Kv:Qp]



× H(β

1

)

−[E:Q]mr1(1−4εd2)(1+2d2δ )Pv∈SΓv.

For the middle term of the right-hand side of (4.24) we have (4.25)

 Y

p∈M (Q) v∈M (K),v|p

max

i

|a

i

|

[Kv v:Qp]



[E:K]

× Y

p∈M (Q) v∈M (K),v|p

 Y

λ∈E,→KK v 1≤h≤m

|(1, λ(β

h

))|

rvh



[Kv:Qp]

=

 Y

p∈M (Q) v∈M (K),v|p

max

i

|a

i

|

[Kv v:Qp]/[K:Q]



[E:Q]

× Y

p∈M (Q) v∈M (K),v|p

 Y

w∈M (E),w|v 1≤h≤m

|(1, β

h

)|

rwh[Ew:Kv]



[Kv:Qp]

=

 Y

v∈M (K)

max

i

ka

i

k

v



[E:Q]

 Y

m

h=1

Y

w∈M (E)

k(1, β

h

)k

rwh



[E:Q]

= H(T )

[E:Q]

 Y

m

h=1

H(β

h

)

rh



[E:Q]

.

Before we estimate the first term of the right-hand side of (4.24) we make some remarks: For each v ∈ S there exists some w

v

∈ M (F ) such that

v

(x)|

v

= |x|

wv

for all x ∈ F , hence |(1, µ

v

v

))|

v

≤ H(α

v

)

[F :Q]/[Fwv:Qp]

. Further from (4.13) we have

Y

p∈M (Q)

16

[E:K]rPv∈M (K),v|pκv[Kv:Qp]

= 16

[E:Q]r

.

Cytaty

Powiązane dokumenty

The proof of (iii) is given in Sections 4 and 5 where we bound the number of polynomials with small and large leading coefficients respectively.. Finally, in Section 6 we complete

Taking the idea from the author’s paper [3] dealing with squarefull in- tegers, we first give a reduction of our problem, which connects (∗) with some exponential sums, but this

(It also states that the 2-class field tower of an arbitrary imaginary quadratic field with four or more prime divisors of discriminant never terminates in the class of CM-fields,

To prove the above theorems we need some general results from the theory of diophantine equations, moreover, several proper- ties of Stirling numbers.. Let K be a finite extension of

Further, in the main case (large p 1 and small q), we are able to get an improvement by removing the restriction that the map constructed is 1-1 (Lemmas 10 and 11).. As for the

Soon after that, employing Vinogradov’s idea, Hua Loo Keng [3] proved that if B is a sufficiently large positive constant and N is sufficiently large, then the even numbers in (2, N

The most famous recent result is in the area of extending P t -sets and is due to Baker and Davenport [1], who used Diophan- tine approximation to show that the P 1 -set {1, 3, 8,

Then, after having transformed the Hecke zeta-functions by means of their functional equation, we apply the summation formula once again, this time “backwards”, in order to